Ch07 Projectile Motion PDF

You might also like

You are on page 1of 34

M th

PROJECTILE MOTION 7
CHAPTER

 projectile
An object thrown into space with certain velocity, fired from a gun or dropped from a moving
plane is called projectile.
A projectile moves with a constant horizontal velocity and at the same time falls freely under
the action of gravity. The path of projectile is called trajectory.

 Trajectory of a Projectile
Let a particle of mass ‘m’ is projected from a point ‘O’ with initial velocity ‘v0’ making an
angle ‘α’ with horizontal. Take ‘O’ as origin and horizontal and vertical lines through ‘O’ as
x-axis and y-axis respectively.

P(x, y)

r
v0

 mg j
α
0 x-axis

Suppose that after time ‘t’ the particle is at point P(x, y) whose position vector is r . i.e.
r = xi + yj
dr dx
= i + j
dy

dt dt dt

v = i + j
dx dy

dt dt

Available At: mathcity.org Contact At: qadri86@yahoo.com


2

dv d2 x

i + 2 j
d2 y
⇒ =
dt dt2 dt

a = 2 i + 2 j

d2 x d2 y
⇒ ________(i)

The gravitational force F acting on the particle at point P(x, y) is


dt dt

F =  mg j ________(ii)
By Newton’s 2nd law of motion

F = ma

= m  2 i + 2 j

d2 x d2 y
________(iii)
dt dt
From (ii) & (iii), we get

m  2 i + 2 j  mg j

d2 x d2 y
dt dt

i + 2 j 0.i  g j

d2 x d2 y

dt2 dt

0 and g
2
d x d2 y

dt2 dt2
On integrating with respect to “t”, we get

=  gt + B
dx dy
= A and _________(iv)
dt dt
Where A & B are constant of integration. To determine the value of these constant we apply
the initial conditions.
Initially at t = 0
dx dy
= v0cosα and = v0sinα
dt dt
Using these values in (iv), we get
A = v0cosα and B = v0sinα
Using values of A & B in (iv), we get

α  gt
dx dy
α and
= v0cosα = v0sinα ________(v)
dt dt
Eq(v) gives the horizontal and vertical components of velocity at any time t.
On integrating (v), with respect to ‘t’, we get

x =
v0cosα t + C and y =
v0sinα t  gt2 + D
1
________(vi)
2
Where C & D are constant of integration. To determine the value of these constant we apply
the initial conditions.
Initially at t = 0, x = 0 and y = 0

Available At: mathcity.org Contact At: qadri86@yahoo.com


3

⇒ C=D=0
Using value of C & D in (vi), we get
x =
v0cosα
α t ________(vii)

y =
v0sinαα t  gt2
1
and ________(viii)
2
Equations (vii) and (viii) are parametric equations of trajectory. Now we find Cartesian
equation of trajectory.
From (vii)
x
t=
v0cosα
Putting value of t in (viii), we get

y =
v0sinα  g
x 1 x 2

v0cosα 2 v0cosα

α
gx2
⇒ y = xtanα sec2 α
2v0 2
Which is Cartesian equation of trajectory of a projectile.

 Vertex, Latus Rectum & Maximum Height of a Projectile


We know that the Cartesian equation of the trajectory of a projectile is:

y = xtanα 
gx2
sec2 α
2v0 2

sec2 α = xtanα  y
gx2

2v0 2

x =   xtanα  y  
2
2v0 2 2v0 2

gsec2 α gsec2 α


2
xv0 2 sinαcosα 2yv0 2 cos2 α
⇒ x =
g g

x2  
2
xv0 sinαcosα 2yv0 2 cos2 α

g g

Adding   on both sides we get


2
v0 2 sinαcosα
g

x  +    
2 2
2
xv0 2 sinαcosα v0 2 sinαcosα v0 2 sinαcosα 2yv0 2 cos2 α
g g g g

⇒ x   = y  
2
v0 2 sinαcosα 2v0 2 cos2 α v0 2 sin2 α
g g 2g
Comparing with (x – h)2 = 4a(y – k), we get

Available At: mathcity.org Contact At: qadri86@yahoo.com


4

h , 4a  , k
v0 2 sinαcosα 2v0 2 cos2 α v0 2 sin2 α
g g 2g

Thus, Vertex =
h, k  , 
v0 2 sinαcosα v0 2 sin2 α
g 2g

Latus Rectum = |4a||


2v0 2 cos2 α
g

Height (H) = k
v0 2 sin2 α
2g

 Focus
X-coordinate of focus = x-coordinate of vertex


v0 2 sinαcosα
g


v0 2 2sinαcosα
2g


v0 2
sin2α
2g

Y-coordinate of focus = H  (Latus Rectum)


1
4

  
v0 2 sin2 α 1 2v0 2 cos2 α
2g 4 g


v0 2 sin2 α v0 2 cos2 α
2g 2g


cos2 α  cos2 α 
v0 2 v0 2
cos2α
2g 2g

Focus =  sin2α, 
v0 2 v0 2
Thus, cos2α
2g 2g

 Equation of Directrix
Height of directrix above the x-axis is:
1
y = H + (Latus Rectum)
4

+  
v0 2 sin2 α 1 2v0 2 cos2 α
2g 4 g

Available At: mathcity.org Contact At: qadri86@yahoo.com


5


v0 2 sin2 α v0 2 cos2 α
+
2g 2g


sin2 α + cos2 α
v0 2
2g


v0 2
2g

 Time of Flight
The time taken by the projectile in reaching the final point is called the time of flight of the
projectile. We know that parametric equation of trajectory of projectile are:
x =
v0cosα t and y =
v0sinα t  gt2
1
2
To find the time of flight put y = 0

v0sinα t  gt2 = 0
1
2

v0sinα  gt t = 0
1

2

v0sinα  gt = 0 t≠0
1

2
2v0sinα
⇒ t=
g
2v0sinα
Thus, T.F =
g

 Range of a Projectile
The range or horizontal range of the projectile is the horizontal distance covered by the
projectile during time of flight.
Range (R) = (Horizontal Velocity)(Time of Flight)


v0cosα   0 sin2α
2v0sinα v2
g g
R will be maximum when sin2α is maximum.
i.e. sin2α = 1
⇒ 2α = sin – 1(1)
⇒ 2α = 900
⇒ α = 450

Available At: mathcity.org Contact At: qadri86@yahoo.com


6

Which shows that if projectile is projected with an angle of 450 then it covers the maximum
horizontal distance.
v0 2
Thus Rmax =
g

 Question 1
Determine the maximum possible range for a projectile fired from a cannon having muzzle
velocity v0 and prove that the height reached in this case is

v0 2
4g

Solution

 Solution
We know that
Rage = (Horizontal Velocity)(Time of Fight)


v0cosα  
2v0sinα v0 2
sin2α
g g
R will be maximum when sin2α is maximum.

i.e. sin2α = 1 ⇒ 2α = sin – 1(1)


⇒ 2α = 900
⇒ α = 450
Which shows that if projectile is projected with an angle of 450 then it covers the maximum
horizontal distance.
v0 2
Thus Rmax =
g
Height reached
v0 2 sin2 α
As
2g
Put α = 45 0

v0 2  
1 2
√2
Height reached
2 2
v0 sin 45 v0 2
2g 2g 4g
 Question 2
What is the maximum range of possible for a projectile fired from a cannon having muzzle
velocity 1mile/sec. What is the height reached in this case.?

Solution

Available At: mathcity.org Contact At: qadri86@yahoo.com


7

We know that
Rage = (Horizontal Velocity)(Time of Fight)


v0cosα  
2v0sinα v0 2
sin2α
g g
R will be maximum when sin2α is maximum.
i.e. sin2α = 1
⇒ 2α = sin – 1(1)
⇒ 2α = 900
⇒ α = 450
v0 2
So Rmax =
g
Given that
v0 = 1 mile/sec = 1760 yard/sec = 1760 × 3 ft/sec = 5280ft/sec
52802
Thus Rmax =
32
871200 feet mile 165 mile
871200
5280
Height reached
v0 2 sin2 α
As
2g
Put α = 45 0

Height reached
v0 2 sin2 45
2g
v0 2  
1 2
√2

v0 2
2g 4g


52802
4(32)
217800 feet

mile 41.25 mile


217800
5280

 Question 3
A cannon has its maximum range R. Prove that
(a) the height reached is R4

(b) the time of flight is 2Rg

Solution

Available At: mathcity.org Contact At: qadri86@yahoo.com


8

We know that
Rage = (Horizontal Velocity)(Time of Fight)


v0cosα  
2v0sinα v0 2
sin2α
g g
R will be maximum when sin2α is maximum.
i.e. sin2α = 1 ⇒ 2α = sin – 1(1) ⇒ 2α = 900 ⇒ α = 450
v0 2
So Rmax =
g
Given that
Rmax = R
v0 2
⇒ R=
g
2
⇒ v0 = Rg

Height reached
v0 2 sin2 α
As
2g

R 
1 2

Rg sin2 45
√2 R
2g 2 4
We know that

Time of Flight
2v0sinα
g
2Rgsin45 2Rg 2R
= = =
g g√2 g

 Question 4
A projectile having horizontal range T, reaches a maximum height H. Prove that it must
have been launched with

(a) an initial speed equal to

gR2 +16H2 

8H
(b) at an angle with horizontal given by

sin1  
4H
R2 +16H2

Solution
We know that
R = Rage = (Horizontal Velocity)(Time of Fight)

Available At: mathcity.org Contact At: qadri86@yahoo.com


9


v0cosα  
2v0sinα v0 2
sin2α
g g

H = Height reached
v0 2 sin2 α
and
2g
Now

R +16H =  sin2α + 16  
2 2
2 2v0 2 v0 2 sin2 α
g 2g
v0 4 2 2
v0 4 sin4 α
= 2 4sin αcos α + 4
g g2
4v0 4
= 2 sin2 αcos2 α + sin2 α
g
4v0 4 sin2 α
= ________(i)
g2
R2 +16H2 4v0 4 sin2 α g
⇒ =
8H g2 4v0 2 sin2 α
v0 2
=

gR +16H2 
g
2
2
⇒ v0 =
8H
gR2 +16H2 
⇒ v0 = 
8H
From (i)

R2 +16H2 =
2v0 2 sinα
g
2
4H 2v0 sin2 α g
R2 +16H2
⇒ = 2
g 2v0 sinα

sinα α sin1  
4H 4H
R2 +16H2 R2 +16H2
⇒ ⇒

 Question 5
Find the range of a rifle bullet when α is the elevation of projection and v0 the speed. Show
that, if the rifle is fired with the same elevation and the speed from a car travelling with
speed V towards the target, and the range will be increased by
α
2v0Vsinα
g

Solution

Available At: mathcity.org Contact At: qadri86@yahoo.com


10

We know that
Rage = (Horizontal Velocity)(Time of Fight)

R
v0cosα  
2v0sinα v0 2
⇒ sin2α
g g
When shell is fired from a car moving with velocity V towards the target then the horizontal
velocity increased by V.
i.e. Horizontal velocity = v0cosα + V
Let R be new range. Then
R = (New Horizontal Velocity)(Time of Fight)


v0cosα + V  
2v0sinα
g


v0 2 2v0Vsinα
sin2α +
g g
Now
Increased in Range = R  R


v0 2 2v0Vsinα v0 2
sin2α + Vsin2α
g g g


2v0Vsinα
g

 Question 6

The range of a rifle bullet is 1200yards when α is the elevation of projection. Show that, if

hour towards the target the range will be increased by 220√tanα


the rifle is fired with the same elevation and the speed from a car travelling at 10 miles per
α feet.

Solution
Given that
R = 1200yards = 1200 × 3 = 3600ft.
and V = 10 mile /h
10 ×1760 ×3
=
3600
44
= ft/sec
3
We know that
Rage = (Horizontal Velocity)(Time of Fight)

Available At: mathcity.org Contact At: qadri86@yahoo.com


11

R
v0cosα  
2v0sinα v0 2
⇒ sin2α
g g

v0 2
Rg

sin2α

v0  
Rg Rg

sin2α 2sinαcosα

When shell is fired from a car moving with velocity V towards the target then the horizontal
velocity increased by V.
i.e. Horizontal velocity = v0cosα + V
Let R be new range. Then
R = (New Horizontal Velocity)(Time of Fight)


v0cosα + V  
2v0sinα
g


v0 2 2v0Vsinα
sin2α +
g g
Now
Increased in Range = R  R


v0 2 2v0Vsinα v0 2
sin2α + Vsin2α
g g g


2v0Vsinα
g


2Vsinα Rg
g 2sinαcosα

V √tanα
g
2R

 √tanα
44 2 × 3600
3 32

√tanα
44 60
3 4
220√tanα

Available At: mathcity.org Contact At: qadri86@yahoo.com


12

 Question 7
A battleship is steaming ahead with sped V and a gun is mounted on the battleship so as
the point straight backwards, and is set at an angle of elevation α. If v0 is the speed of
projection (relative to the gun), show that the range is
α
v0 cosα
α  V
2v0
sinα
g
Also prove that the angle of elevation for maximum range is

V +V2 + 8v0 2
cos 1 " $

! #
4v0

Solution
When shell is fired from a battleship moving ahead with velocity V towards the target which
is behind the battleship then the horizontal velocity decreased by V.
i.e. Horizontal velocity = v0cosα  V
Let R be the range. Then
R = (Horizontal Velocity)(Time of Fight)


v0cosα  V  
2v0sinα
g

v0cosα  V
2v0sinα
g
Differentiating w.r.t “α”, we get

%cosα
v0cosα  V & sinα
v0sinα '
dR 2v0
dα g

(v0cos2 α  Vcosα  v0sin2 α)


2v0
g

(v0cos2 αsin2 α  Vcosα)


2v0
g

*v0 cos2 α 
1 cos2 α  Vcosα+
2v0
g

%v0
2cos2 α  1  Vcosα'
2v0
g

%2v0cos2 α  Vcosα  v0'


2v0
g
Differentiating again w.r.t “α”, we get

%4v0cosαsinα + Vsinα'
d2 R 2v0
dα2 g

Available At: mathcity.org Contact At: qadri86@yahoo.com


13

sinα%V  4v0cosα'
2v0
g
dR
Putting = 0, we get

%2v0cos2 α  Vcosα  v0' 0


2v0
g
⇒ 2v0cos2 α  Vcosα  v0 0

V ± V2 + 8v02
⇒ cosα =
4v0
V + V2 + 8v02
At cosα =
4v0

V + V2 + 8v02
sinα -V  4v0 .
d2 R 2v0
dα2 g 4v0

sinα *V2 + 8v02 +


2v0
g

d2 R V + V2 + 8v02
So < 0 at cosα =
dα2 4v0
V + V2 + 8v02
Which shows that R is maximum at cosα = .
4v0
Thus the angle of elevation for maximum range is given by

V + V2 + 8v02
cos 1
0 1
4v0

 Question 8

speeds up to 80feet per seconds. Prove that a man 100 feet away is in danger for 5
A shell bursts on contact with the ground and pieces from it fly in all directions with all

√2
seconds.

Solution
We know that
Rage = (Horizontal Velocity)(Time of Fight)


v0cosα  
2v0sinα
g


v0 2
sin2α
g

Available At: mathcity.org Contact At: qadri86@yahoo.com


14

Given that

R = 100ft, v0 = 80ft/sec and g = 32ft/sec2

802
So 100 = sin2α
32
6400
⇒ 100 = sin2α
32

⇒ 2α = sin1   ⇒ 2α = 30, 150 ⇒ α = 15, 75


1 1
⇒ sin2α =
2 2
For the range of 100ft. there are two angles of projection. Let T1 and T2 be the times of the
flights respectively. Then

2v0sin15 2v0sin75
T1 = and T2 =
g g

Let T be the maximum time of danger for the man. Then


T = T2 – T1


2v0sin75 2v0sin15 T2
=
g g

sin75  sin15
2v0
=
75  15
g
2cos   sin  
2v0 75 + 15
= 750
g 2 2 T1
2v0
= 2cos45sin30 150
g 100ft.
  =
4(80) 1 1 5
32 √2 2 √2
= sec

 Question 9
A number of particles are projected from the same point at the same instant in various
directions with speed v0. Prove that at any subsequent time t, they will be on a sphere of
radius v0t and determine the motion of the centre of the sphere.

Solution
Let a particle moving with velocity v0 makes an angle α. Let after time ‘t’ a particle is at a
point P(x, y, z). Then
x = (v0cosα)t
1 2 1
y = (v0sinα) t – gt ⇒ y + gt2 = (v0sinα) t
2 2

Available At: mathcity.org Contact At: qadri86@yahoo.com


15

z=0
Squaring and adding we get

x +  y + gt  + z =
(v0cosα)t 2 +
(v0sinα)t 2
2
1 2 2 2
2
= v0 2 t2 cos2 α + sin2 α
=
v0 t 2

Which is a sphere of a radius v0t centered at 0,  gt2, 0 . Since the centre lies on the
1
2
vertical axis and as t increases centre descends under gravity along vertical axes.

 Question 10
Prove that the speed required to project a particle from a height h to fall a horizontal
distance a from the point of projection is at least

g a2 + h2  h

Solution

v0

α
x
O a
h

P(a, h)

Let O be the point of the projection from where the projectile is projected. Let v0 be the
velocity making angle α with horizontal. Let h be the height of the point of the projection O

are (a,  h).


and projectile fall a distance a from O. Let it falls at a point P, therefore the coordinates of P

We know that the Cartesian equation of the trajectory of a projectile is:

y = xtanα 
gx2
sec2 α
2v0 2
Since P(a,  h) lies on it, therefore

 h = atanα 
ga2
sec2 α
2v0 2
⇒  2v0 2 h = 2av0 2 tanα  ga2 1 & tan2 α

Available At: mathcity.org Contact At: qadri86@yahoo.com


16

⇒ ga2 tan2 α  2v0 2 xtanα + ga2  2v0 2 h = 0


Since it is coodratic in tanα and tanα is real therefore discriminate is greater than zero.
i.e.
b2 – 4ac ≥ 0

2av0 2 3  4
ga2
ga2  2v0 2 h ≥ 0

2av0 2 3 ≥ 4
ga2
ga2  2v0 2 h

4a2 v0 ≥ 4g2 a4  8gha2 v0 v0 4 ≥ g2 a2  2ghv0 2


4 2
⇒ ⇒
⇒ v0 4 + 2ghv0 2 ≥ g2 a2 ⇒ v0 4 + 2ghv0 2 +
gh 2 ≥ g2 a2 +
gh 2

v0 2 + gh 3 ≥ g2 a2 & h2 

⇒ v0 2 + gh ≥ g2 a2 & h2 

⇒ v0 2 ≥ ga2 & h2  gh

⇒ v0 ≥ g a2 & h2  h

Hence the least velocity of projection is

v0 = g a2 & h2  h

 Question 11
A projectile is launched at an angle α from a cliff of height H above the see level. if it falls
into the sea at a distance D from the base of the cliff, prove that the maximum height above
sea level is

D2 tan2 α
α
H+
4
H + Dtanα

Solution

v0 h

α
x
O D
H

P(D, H)

Available At: mathcity.org Contact At: qadri86@yahoo.com


17

Let O be the point of the projection from where the projectile is projected. Let v0 be the
velocity making angle α with horizontal. Let H be the height of the point of the projection O

are (D,  H).


and projectile fall a distance D from O. Let it falls at a point P, therefore the coordinates of P

Let h be the height above the x-axis then


v0 2 sin2 α
h= ________(i)
2g
We know that the Cartesian equation of the trajectory of a projectile is:

y = xtanα 
gx2
sec2 α
2v0 2
Since P(D,  H) lies on it, therefore

 H = Dtanα 
gD2
sec2 α
2v0 2
gD2
⇒ Dtanα + H = sec2 α
2v0 2

2
gD2
⇒ v0 =
2
H + Dtanα cos2 α
Using value of v0 2 in (i), we get

h=  
gD2 sin2 α
2
H + Dtanα cos2 α 2g

=  
D2 tan2 α
4
H + Dtanα
Height above sea level = H + h

H+
D2 tan2 α
4
H + Dtanα

 Question 12
A ball is dropped from the top of a tower of height h. At the same moment, another ball is
thrown from a point of the ground at a distance k from the foot of tower so as to strike the
first ball at the depth d. Show that the initial speed and the direction of projection of the
speed ball are respectively

gh2  k2 
 and tan 1  
h
2d k

Solution

Available At: mathcity.org Contact At: qadri86@yahoo.com


18

I
d
P

h–d h

v0
α
x
II O k

Let the first ball is dropped from the height h and it strikes the second ball at the depth d at a
point P whose coordinates are (k, h – d).
We know that
1 2
x = ut + gt
2
For 1st ball, x = d and u = 0
1 2
So d= gt ________(i)
2
For 2nd ball parametric equations are
x = (v0cosα)t

and y = (v0sinα)t 
1 2
gt
2
Since P(k, h – d) lies on it therefore
k = (v0cosα)t ______(ii)

and h  d = (v0sinα)t 
1 2
gt ______(iii)
2
Adding (i) and (iii), we get
h = (v0sinα)t ______(iv)
Squaring (ii) and (iv) then adding, we get
h2 + k2 =
(v0sinα)t 2 +
(v0cosα)t 2

= v02 t
sin2 α + cos2 α = v02 t2
2

2d
= v0 2 By(i)
g

gh2 + k2  gh2 + k2 
⇒ v0 2
⇒ v0 
2d 2d

Available At: mathcity.org Contact At: qadri86@yahoo.com


19

From (ii) & (iv), we have

α = tan1  
h h
tanα = ⇒
k k

 Question 13
From a gun placed on a horizontal plane, which can fire a shell with speed 2gH, it is
required to thro a shell over a wall of height h, and the elevation of the gun cannot exceed
α < 450. Show that this will be possible only when h < Hsin2α, and that, if this condition
be satisfied, the gun must be fired from within a strip of the plane whose breadth is
α H
Hsin2 α  h
4cosα

Solution
y

C D

v0
h h

α
x
O O A B

Let AC be a wall of height h and particle be projected at O with speed v0 making an angle α.
Then v0 = 2gH (given)
Now for a shell to cross the wall, the height of the wall is less than the height of vertex.
v0 2 sin2 α
i.e. h <
2g

 v0 = 2gH
2gHsin2 α
⇒ h <
2g
⇒ h < Hsin2 α
Which is required.
We know that the Cartesian equation of the trajectory of a projectile is:

y = xtanα 
gx2
sec2 α
2v0 2
Putting y = h and v0 = 2gH, we get

Available At: mathcity.org Contact At: qadri86@yahoo.com


20

h = xtanα 
x2
sec2 α

4hH = 4xHtanα  x2 sec2 α


4H

⇒ x2 sec2 α  4xHtanα + 4hH 0
⇒ x2  4xHtanαcos2 α + 4hHcos2 α 0
⇒ x2  4xHsinαcosα + 4hHcos2 α 0

4Hsinαcosα ±16H2 sin2 α cos2 α  4


1 (4hHcos2 α)
⇒ x=
2
4Hsinαcosα ±16H2 sin2 α cos2 α  16hHcos2 α
=
2
4Hsinαcosα ± 4cosαH2 sin2 α  hH
=
2
= 2Hsinαcosα ± 2cosαH2 sin2 α  hH

Thus

OB = 2Hsinαcosα + 2cosαH2 sin2 α  hH

OA =2Hsinαcosα  2cosαH2 sin2 α  hH


Hence the breadth of the strip is:

OO = AB = OB – OA = 4cosαH2 sin2 α  hH

Available At: mathcity.org Contact At: qadri86@yahoo.com


21

 Question 14
A shell fired with speed V at an elevation θ, hits an airship at height H,, which is moving
horizontally away from the gun with speed v0. Show that, if

2Vcosθ
θ  v0 V2 sin2 θ  2gH = v0 Vsinθ
θ

The shell might also have hit the air ship if the latter had remained stationary in the position it
occupied when the gun was actually fired.

Solution
y

A B

v
H H

α
x
O

Let A be the position of airship when shot was fired and it hit plane at B. If t is the time taken
by hell to reach height H.
We know that

y = (v0sinα)t 
1 2
gt
2
Putting v0 = V, θ = α and y = H, we get

H = (Vsinθ)t 
1 2
gt

2H = (Vsinθ)t  gt2
2

⇒ gt2  (Vsinθ)t & 2H = 0

2Vsinθ ±4V2 sin2 θ  8Hg


⇒ t=
2g

Vsinθ ±V2 sin2 θ  2Hg


=
g

Available At: mathcity.org Contact At: qadri86@yahoo.com


22

Let t1 and t2 be the time taken by the shell to reach the point A and B respectively. Then

Vsinθ  V2 sin2 θ  2Hg Vsinθ +V2 sin2 θ  2Hg


t1 = and t2 =
g g
Let T be the time taken by the shell from A to B. Then
T = t2 – t1

Vsinθ +V2 sin2 θ  2Hg Vsinθ  V2 sin2 θ  2Hg



g g

2V2 sin2 θ  2Hg



g
Now |AB| = Distance covered by shell
= (Horizontal Velocity)(Time)

2V2 sin2 θ  2Hg



Vcosθ
g
2VcosθV2 sin2 θ  2Hg
______(i)
g
Now |AB| = Distance covered by airship
= (Velocity)(Time) = v0t2

v0 Vsinθ +V2 sin2 θ  2Hg


______(ii)
g
From (i) and (ii), we get

2VcosθV2 sin2 θ  2Hg v0 Vsinθ +V2 sin2 θ  2Hg



g g

⇒ 2VcosθV2 sin2 θ  2Hg v0 Vsinθ + v0 V2 sin2 θ  2Hg

⇒ v0 Vsinθ = 2VcosθV2 sin2 θ  2Hg  v0 V2 sin2 θ  2Hg

⇒ v0 Vsinθ =
2Vcosθ  v0 V2 sin2 θ  2Hg

Which is required.

Available At: mathcity.org Contact At: qadri86@yahoo.com


23

 Question 15
An aeroplane is flying with constant speed v0 and at constant height h. Show that if, a gun
is fired point blank at the aeroplane after it has passed directly over the gun when its angle
of elevation as seen from the gun is α, the shell will hit the aero plane provided that
2

Vcosα
α  v0 v0 tan2 α = gh
Where V is the initial speed of the shot, the path being assumed parabolic.

Solution

A B

α
O C D
Let A be the position of plane when shot was fired and it hit plane at B. Let v0 be the speed of
plane.
From fig.
AB = v0t (  S = vt)
Horizontal coordinate of B = OD
= OC + CD
= OC + AB  AB = CD
= OC + v0t  AB = v0t _____(i)
In ∆AOC
AC
= tanα
OC

= tanα ⇒ OC ⇒ OC hcotα
h h

OC tanα
Using value of OC in (i), we get
Horizontal coordinate of B = hcotα + v0t
Thus coordinates of B are (hcotα + v0t, h)
The parametric equations are:
x = (v0cosα)t

y = (v0sinα)t 
1 2
gt
2

Available At: mathcity.org Contact At: qadri86@yahoo.com


24

Here v0 = V therefore
x = (Vcosα)t ______(ii)

y = (Vsinα)t 
1 2
gt ______(iii)
2
Since B lies on the trajectory therefore x = hcotα + v0t and y = h
Using x = hcotα + v0t in (ii), we get
hcotα + v0t = (Vcosα)t
⇒ hcotα = (Vcosα)t  v0t
⇒ hcotα = (Vcosα  v0)t

Vcosα  v0
hcotα
⇒ t=

Using y = h in (iii), we get

h = (Vsinα)t 
1 2
gt
2

= (Vsinα) Vcosα  v  g Vcosα  v


hcotα 1 hcotα 2

0 2 0

=   g 
2

Vcosα  v0 2 Vcosα  v0
Vhcosα 1 hcotα

1 =   g 
Vcosα  v0 2 Vcosα  v0
2
Vcosα h cotα

⇒ 2
Vcosα  v0 2 = 2
Vcosα
Vcosα  v0  hgcot2 α

⇒ ghcot2 α = 2
Vcosα
Vcosα  v0  2
Vcosα  v0 2

⇒ ghcot2 α = 2
Vcosα  v0
Vcosα  Vcosα + v0

⇒ gh = 2v0
Vcosα  v0 tan2 α

Available At: mathcity.org Contact At: qadri86@yahoo.com


25

 Parabola of Safety
A parabola which touches the every trajectory of a projectile which is formed inward for
different value of angle of projection with same initial velocity v0 is called parabola of safety.
y

Parabola of safety

x
O
Equation of trajectory of a parabola is:

y = xtanα 
gx2
sec2 α
2v0 2

y = xtanα 
1 + tan2 α
gx2

2v0 2

y = xtanα  
gx2 gx2
⇒ tan2 α
2v0 2 2v0 2

tan α  xtanα +  2 + y = 0
gx2 2
gx2

2v0 2 2v0
This equation is quadratic in tanα. For envelope put discriminate of equation equal to zero.
i.e.
b2 – 4ac = 0

x2  4  2   2 + y = 0
gx2 gx2

2v0 2v0

1  2  2  = 0
g gx2 + 2v0 2 y

v0 2v0 2

g = 1
gx2 + 2v0 2 y

v0 4
2 2
v0 4
⇒ gx + 2v0 y =
g

gx2 =  2v0 2 y +
v0 4

g

Available At: mathcity.org Contact At: qadri86@yahoo.com


26

x = 
2
2v0 2 y 2v0 4
⇒ +
g 2g2

x =  y  
2
2v0 2 v0 2

g 2g
Which is the equation of parabola of safety.

 Question 16

2ga of which the vertical component is V. Show that at time t = 2a/V the particle is on a
A particle is projected at time t = 0 in a fixed vertical plane from a given point O with speed

fixed parabola (parabola of safety), that its path touches the parabola, and that its direction of
motion is then perpendicular to its direction of projection.

Solution

Given that v0 = 2ga and V = v0sinα

We know that the equation of parabola of safety is:

x =  y  
2
2v0 2 v0 2
g 2g

2ga
x =  7y  8
2 2
2
22ga

g 2g

=  4a
y  a _______(i)
Which is the equation of the parabola of safety.
Let P(x, y) be a point on trajectory then
x = (v0cosα)t ______(ii)

y = (v0sinα)t 
1 2
gt ______(iii)
2

ii becomes
2a
At t =
V
2a
x = (v0cosα)
V
2a
= (v0cosα)
v0sinα
⇒ x = 2acotα ______(iv)


iii becomes
2a
At t =
V

Available At: mathcity.org Contact At: qadri86@yahoo.com


27

 g 
2a 1 2a 2
y = (v0sinα)
V 2 V

 g 
2a 1 2a 2
= (v0sinα)
v0sinα 2 v0sinα

= 2a  g  
2
1 2a
2 2ga sinα

= 2a  a cosec 2 α ______(v)
Thus the coordinates of P are (2acotα, 2a  a cosec 2 α)
Putting (iv) and (v) in (i), we get

2acotα 2 =  4a
2a  a cosec 2 α  a
⇒ 4a2 cot2 α =  4a
a  acosec 2 α
⇒ a2 cot2 α = a2
cosec 2 α  1
⇒ a2 cot2 α = a2 cot2 α
⇒ L.H.S = R.H.S
Thus P(x, y) lies on the parabola of safety. So at t = 2a/V this trajectory touches the parabola
of safety.
Differentiate (ii) & (iii) w.r.t “t”, we get

= v0 sinα  gt
dx dy
= v0 cosα and
dt dt

= :
dy dy dx
Now

v0 sinα  gt
dx dt dt

=
v0 cosα
2a
At t =
V

dy v0 sinα  g V
2a
=
dx v0 cosα
Vv0 sinα  2ag
=
Vv0 cosα

v0 sinα v0 sinα  2ag
 V = v0 sinα

v0 sinα v0 cosα
=

2gasinα2gasinα  2ag
 v0 = 2ga
2gasinα2gacosα
=

Available At: mathcity.org Contact At: qadri86@yahoo.com


28

2agsin2 α  2ag
=
2agsinαcosα
1  sin2 α
= 
sinαcosα

= 
cos2 α
sinαcosα
=  cotα
This is the slope of the tangent at t = 2a/V

= tanθ
dy
Since
dx
Where θ is inclination of the tangent at P(x, y). Then
tanθ =  cotα tan(900 + α)
⇒ θ = 900 + α
Thus at t = 2a/V, its direction of motion is perpendicular to the direction of projection.

 Range of a Projectile on Inclined Plane

Let a plane be inclined at an angle β to the horizontal. Let a particle is projected from point O
with velocity v0 by making an angle α to the horizontal with β < α. Let the projectile meet
the inclined plane at a point P(x, y). Then OP = r is called the range of projectile on inclined
plane.
Then x = rcosβ and y = rsinβ
So P(x, y) = (rcosβ, rsinβ)

P(x, y)

v0 r
rsinβ
α
β
x
O rcosβ

Equation of the trajectory is:

y = xtanα 
gx2
sec2 α
2v0 2

Available At: mathcity.org Contact At: qadri86@yahoo.com


29

Since P lies on it therefore

rsinβ = rcosβtanα 
gr2 cos2 β 2
sec α
2v0 2

sinβ = cosβtanα 
grcos2 β

2v0 2 cos2 α

= cosβtanα  sinβ
grcos2 β

2v0 2 cos2 α

 sinβ
grcos2 β sinα
⇒ = cosβ
2v0 2 cos2 α cosα
grcos2 β sinαcosβ  cosαsinβ
⇒ =
2v0 2 cos2 α cosα

= sin
α  β
grcos2 β

2v0 2 cosα

cosαsin
α  β
2v0 2
⇒ r= ____________(i)
gcos2 β
Since sin(α + β)  sin(α  β) = 2cosαsinβ
Replacing β by α  β, we get
sin(2α  β)  sin(β) = 2cosαsin(α  β) ___________(ii)
From (i) and (ii), we get
v0 2 sin(2α  β)  sinβ
r=
g cos2 β
Which is the range of a projectile on an inclined plane.
Range will maximum if sin(2α  β) = 1
⇒ 2α  β = sin – 1(1)
⇒ 2α = β + 900
β
⇒ α = + 450
2

v0 2 1  sinβ
Thus,

r<=> =  
g cos2 β
v0 2 1  sinβ
 
g 1  sin2 β
=

1  sinβ
 =
g
1  sinβ
1 + sinβ
v0 2 v0 2
=
g
1 + sinβ

Available At: mathcity.org Contact At: qadri86@yahoo.com


30

 Question 17

2gk, and the guns in the fort are at a height h above the guns in the ship. If d1 and d2 are
A fort and a ship are both armed with guns which give their projectiles a muzzle velocity

the greatest horizontal ranges at which the fort and ship, respectively, can engage, prove
that

=
kh
d1 k+h
d2

Solution

r
h

v0

α β
x
S d2 A

Let S be ship and F be fort. Let fort makes angle β with x-axis. i.e. ∠ASF = β. Let SF = r
Since d2 is greatest horizontal range for gun in the ship so r is the maximum range on inclined
plane with inclination β. Then
v0 2
r=
g
1 + sinβ

Put v0 = 2gk
2gk
⇒ r=
g
1 + sinβ
⇒ 2k r + rsinβ
From fig.
h = rsinβ
⇒ 2k r + h
⇒ r 2k – h
Also from fig.
r2 = h2 + d2 2

Available At: mathcity.org Contact At: qadri86@yahoo.com


31


2k – h 2 = h2 + d2 2

⇒ 4k2 + h2  4hk = h2 + d2 2
⇒ d2 2 4k2  4hk 4k
k  h _______(i)
For a gun in fort, change h to – h
d1 2 4k
k + h _______(ii)
From (i) and (ii), we get

4k
k + h

d1 2
d2 2 4k
k  h

k+h

kh
d1

d2

 Question 18
A shell of mass m1 + m2 is fired with a velocity whose horizontal and vertical components
are u, v and at the highest point in its path the shell explodes into two fragments m1, m2.
The explosion produces additional kinetic energy E, and the fragments separate in a
horizontal direction. Show that they strike the ground at a distance apart which is equal to

2E   
V 1 1
g m1 m2

Solution
Let v0 be the velocity of the projection. then by given conditions
u = v0cosα _______(i)
v = v0sinα ______(ii)
At the highest point, there is only horizontal velocity u. Let v1 and v2 be the velocities of m1
and m2 respectively at the time of explosion. Then by law of conservation of momentum.
m1v1 + m2v2 = (m1 + m2)u
m 1 v1 + m 2 v2
⇒ u= _______(iii)
m1 + m2
Now
Increase in K.E. = K.E. after explosion – K.E. before explosion

E =  m1 v1 2 + m2 v2 2   
m1 + m2 u2 
1 1 1

2E = m1 v1 2 + m2 v2 2 
m1 + m2 u2
2 2 2

Available At: mathcity.org Contact At: qadri86@yahoo.com


32


m1 + m2  
2 2
m 1 v1 + m 2 v2 2
⇒ 2E = m1 v1 + m2 v2 By (iii)

2
m1 + m2 E =
m1 + m2
m1 v1 2 + m2 v2 2 
m1v1 + m2v2 2
m1 + m2

= m 1 2 v1 2 + m 1 m 2 v2 2 + m 2 m 1 v1 2 + m 2 2 v2 2  m 1 2 v1 2  m 2 2 v2 2
 2m1 m2 v1 v2
= m1 m2 v2 2 + m2 m1 v1 2  2m1 m2 v1 v2
= m1 m2
v2 2 + v1 2  2v1 v2
= m1 m2
v2  v1 3
2
m1 + m2 E

v2  v1 3
m1 m2

v2  v1 2E   
1 1

m1 m2

Which is relative velocity of m1 + m2.


Now the time taken by pieces to touch the ground is given by:
1
Time = (Time of flight)
2

=  =
1 2v0 sinα v0 sinα
2 g g
v
= By (ii)
g
Distance Apart = (Relative velocity)(Time)


v2  v1
v
g

2E   
v 1 1
g m1 m2

Which is required.

Available At: mathcity.org Contact At: qadri86@yahoo.com


33

 Question 19 (Speed of projectile)


Show that least speed with which a particle must be projected so that it passes through two
points P and Q at height hP and hQ respectively is:

ghP + hQ + PQ

Solution
y
M N Directrix

v0 2
P Q y=
2g

hP hQ
S

x
O L R

Let S be the focus.


From fig.
LM = LP + PM
v0 2
⇒ = hP + PM _________(i)
2g
RN = RQ + QN
v0 2
⇒ = hQ + QN _________(ii)
2g
Adding (i) and (ii), we get
v0 2
= hP + hQ + PM + QN
g
⇒ v0 2 = ghP + hQ + PM + QN
= ghP + hQ + PS + QS By focus-directrix property

Available At: mathcity.org Contact At: qadri86@yahoo.com


34

⇒ v0 = ghP + hQ + PS + QS

v0 is least when PS + QS is least, which is least when S lies on PQ. i.e. when
PS + QS = PQ
Hence


v0 min = ghP + hQ + PQ

%%%%% End of The Chapter # 7 %%%%%

Available At: mathcity.org Contact At: qadri86@yahoo.com

You might also like